LSAT and Law School Admissions Forum

Get expert LSAT preparation and law school admissions advice from PowerScore Test Preparation.

 yrivers
  • Posts: 68
  • Joined: Mar 15, 2017
|
#33543
Hi,

Could someone provide explanation for this question, please? An explanation of why the others are incorrect would be helpful.

I chose answer A Looking at the answer key (D is the correct answer), is it because the entire population could've gotten much bigger during the party's administration, making the unemployment rate drop (and not due to the party's policies)? Option D sounded too certain/specific to me: no where in the stimulus could I find where the evidence cited in support actually provides more support for the denial of the conclusion.

From reading the LRB, we know that with a number and percentage, we can't tell if the actual unemployment went up or down (total population could've dropped or grown).

Please let me know.

Thanks!
Yaesul
 Robert Carroll
PowerScore Staff
  • PowerScore Staff
  • Posts: 1787
  • Joined: Dec 06, 2013
|
#33564
Yaesul,

As this is a Flaw in the Reasoning question, the problem is fully contained in the stimulus as written. Here, the problem is the disconnect between the politican's premises and conclusion. The conclusion is that there are fewer unemployed people. This is a claim about a number, not a percent. Now look at the premise that's supposed to prove this: the politician says that the number of people unemployed increased. Ignore the percent for now, because it's not actually relevant - the premise says that the number of people unemployed increased, by the conclusion says that the number decreased! The politician's premise actually supports the opposite of the politician's conclusion.

The tricky aspect of it is the percent, which, as I said, is not relevant. Imagine a hypothetical situation. There were 100 unemployed four years ago. Now there are 119 unemployed. The number of unemployed increased by less than 20%. But...it still increased! So of course it didn't go down. The conclusion was all about numbers, not the rate, so this is where the politician went wrong.

The unemployment rate was never relevant here because it wasn't discussed. The conclusion was all about how many actual people were unemployed, which is where the flaw was, as I've shown above.

As far as the other answers - it's a Flaw question, so the answer must describe a flaw that already happened in the stimulus.

Answer choice (A) is not the flaw because the opposition's claims were not relevant, so we don't need to know what they are. Thus, the politician did not do anything wrong by not specifying them.

Answer choice (B) is not the flaw because the uniformity of unemployment is not relevant. Thus, it's not a flaw for the politician to have failed to prove anything about uniformity.

Answer choice (C) is not the flaw because the politician's argument was not about why unemployment has changed, but simply how it's changed (and, as we've seen, it hasn't changed the way the politician thinks it has).

Answer choice (E) is not the flaw because, as before, the argument is about the numbers, not what factors may affect unemployment.

Robert Carroll
User avatar
 simonsap
  • Posts: 34
  • Joined: Jun 14, 2021
|
#87908
eg. scenario:

From 1996-2000, liberals in power. Unemployment went up 20%.
From 2000-2004, conservatives in power. Unemployment went up 15% (Unemployment ROSE by less than 20% -- it still ROSE)

Therefore, assuming a static population, EVEN MORE people became unemployed - unemployment didn't go down.
The conclusion that fewer people are now unemployed is just wrong.

100 people UE. 20% increase from base = 120 people UE.
120 people UE. additional 15% increase from base (20+15) = 135 people UE.

Get the most out of your LSAT Prep Plus subscription.

Analyze and track your performance with our Testing and Analytics Package.